How to do $sum_{k=1}^{infty} frac{cos(kx)}{k^2}$?












2












$begingroup$


I'm a physicist with no clue how to calculate $S = sum_{k=1}^{infty} frac{cos(kx)}{k^2}$.



One handbook says the answer is $frac{1}{12}(3x^2 - 6 pi x + 2 pi^2)$ on the interval $0 leq x leq 2pi$, which would give the value everywhere since the sum is periodic in $2pi$.



Altland's Condensed Matter Field Theory gives the value as



$frac{pi^2}{6} - frac{pi |x|}{2} + frac{x^2}{4} + dots$, giving no domain and with a weird absolute value, implying the series is valid everywhere and has higher powers. But how can this be correct? The series is periodic and those first three terms give the complete answer on $0 leq x leq 2pi$, so trying to define the result everywhere with an infinite power series seems senseless to me.



Anyway, can anybody shed light on how to perform the summation? I've tried writing $S$ as a contour integration, like so:



$frac{1}{2pi i} oint dz g(z) frac{cos(i z x)}{-z^2} $, where $g(z) = frac{beta}{exp(beta z) - 1}$, a counting function with simple poles at $z = i k$, $k = ..., -2, -1, 0, 1, 2, dots$, and the contour contains all the poles of $g(z)$ for $k = 1, 2, 3, ...$.



Now, this is not my expertise, but I want to learn. The trick now is to pick a different contour (possibly going off to infinity), such that the integral can be performed. I see that the product $g(z) frac{cos(i z x)}{-z^2}$ goes to zero at infinity, but I cannot see how to deform the contour such that the integral becomes tractable.










share|cite|improve this question











$endgroup$








  • 1




    $begingroup$
    "trying to define the result everywhere with an infinite power series seems senseless to me" It does make sense. For instance, $cos(x)$ itself is defined on the entire number line as $$ cos(x) = 1-frac{x^2}2 + frac{x^4}{24} - frac{x^6}{720} + cdots $$As to whether the two solutions you have there coincide, and how the power series is supposed to continue, I do not know.
    $endgroup$
    – Arthur
    Jan 25 '16 at 10:13










  • $begingroup$
    The best is to learn the theory of Bernoulli polynomials. These are periodic function of period $2pi$ whose restriction to the interval $[0,2pi]$ is a polynomial.
    $endgroup$
    – Thomas
    Jan 25 '16 at 10:27










  • $begingroup$
    Mathematica gives $$S=frac{text{Li}_2left(e^{i x}right)+text{Li}_2left(e^{-i x}right)}{2}$$ which isn't really enlightening given what generated it.
    $endgroup$
    – GPhys
    Jan 25 '16 at 10:33












  • $begingroup$
    This is just a Fourier series of a well known function. If you go in reverse, it's quite obvious, just compute the fourier coefficients of $ax^2+bx+c$. How to compute the sum without knowing the answer? I'll find the shortest way and post it below if noone else does.
    $endgroup$
    – orion
    Jan 25 '16 at 11:21
















2












$begingroup$


I'm a physicist with no clue how to calculate $S = sum_{k=1}^{infty} frac{cos(kx)}{k^2}$.



One handbook says the answer is $frac{1}{12}(3x^2 - 6 pi x + 2 pi^2)$ on the interval $0 leq x leq 2pi$, which would give the value everywhere since the sum is periodic in $2pi$.



Altland's Condensed Matter Field Theory gives the value as



$frac{pi^2}{6} - frac{pi |x|}{2} + frac{x^2}{4} + dots$, giving no domain and with a weird absolute value, implying the series is valid everywhere and has higher powers. But how can this be correct? The series is periodic and those first three terms give the complete answer on $0 leq x leq 2pi$, so trying to define the result everywhere with an infinite power series seems senseless to me.



Anyway, can anybody shed light on how to perform the summation? I've tried writing $S$ as a contour integration, like so:



$frac{1}{2pi i} oint dz g(z) frac{cos(i z x)}{-z^2} $, where $g(z) = frac{beta}{exp(beta z) - 1}$, a counting function with simple poles at $z = i k$, $k = ..., -2, -1, 0, 1, 2, dots$, and the contour contains all the poles of $g(z)$ for $k = 1, 2, 3, ...$.



Now, this is not my expertise, but I want to learn. The trick now is to pick a different contour (possibly going off to infinity), such that the integral can be performed. I see that the product $g(z) frac{cos(i z x)}{-z^2}$ goes to zero at infinity, but I cannot see how to deform the contour such that the integral becomes tractable.










share|cite|improve this question











$endgroup$








  • 1




    $begingroup$
    "trying to define the result everywhere with an infinite power series seems senseless to me" It does make sense. For instance, $cos(x)$ itself is defined on the entire number line as $$ cos(x) = 1-frac{x^2}2 + frac{x^4}{24} - frac{x^6}{720} + cdots $$As to whether the two solutions you have there coincide, and how the power series is supposed to continue, I do not know.
    $endgroup$
    – Arthur
    Jan 25 '16 at 10:13










  • $begingroup$
    The best is to learn the theory of Bernoulli polynomials. These are periodic function of period $2pi$ whose restriction to the interval $[0,2pi]$ is a polynomial.
    $endgroup$
    – Thomas
    Jan 25 '16 at 10:27










  • $begingroup$
    Mathematica gives $$S=frac{text{Li}_2left(e^{i x}right)+text{Li}_2left(e^{-i x}right)}{2}$$ which isn't really enlightening given what generated it.
    $endgroup$
    – GPhys
    Jan 25 '16 at 10:33












  • $begingroup$
    This is just a Fourier series of a well known function. If you go in reverse, it's quite obvious, just compute the fourier coefficients of $ax^2+bx+c$. How to compute the sum without knowing the answer? I'll find the shortest way and post it below if noone else does.
    $endgroup$
    – orion
    Jan 25 '16 at 11:21














2












2








2


2



$begingroup$


I'm a physicist with no clue how to calculate $S = sum_{k=1}^{infty} frac{cos(kx)}{k^2}$.



One handbook says the answer is $frac{1}{12}(3x^2 - 6 pi x + 2 pi^2)$ on the interval $0 leq x leq 2pi$, which would give the value everywhere since the sum is periodic in $2pi$.



Altland's Condensed Matter Field Theory gives the value as



$frac{pi^2}{6} - frac{pi |x|}{2} + frac{x^2}{4} + dots$, giving no domain and with a weird absolute value, implying the series is valid everywhere and has higher powers. But how can this be correct? The series is periodic and those first three terms give the complete answer on $0 leq x leq 2pi$, so trying to define the result everywhere with an infinite power series seems senseless to me.



Anyway, can anybody shed light on how to perform the summation? I've tried writing $S$ as a contour integration, like so:



$frac{1}{2pi i} oint dz g(z) frac{cos(i z x)}{-z^2} $, where $g(z) = frac{beta}{exp(beta z) - 1}$, a counting function with simple poles at $z = i k$, $k = ..., -2, -1, 0, 1, 2, dots$, and the contour contains all the poles of $g(z)$ for $k = 1, 2, 3, ...$.



Now, this is not my expertise, but I want to learn. The trick now is to pick a different contour (possibly going off to infinity), such that the integral can be performed. I see that the product $g(z) frac{cos(i z x)}{-z^2}$ goes to zero at infinity, but I cannot see how to deform the contour such that the integral becomes tractable.










share|cite|improve this question











$endgroup$




I'm a physicist with no clue how to calculate $S = sum_{k=1}^{infty} frac{cos(kx)}{k^2}$.



One handbook says the answer is $frac{1}{12}(3x^2 - 6 pi x + 2 pi^2)$ on the interval $0 leq x leq 2pi$, which would give the value everywhere since the sum is periodic in $2pi$.



Altland's Condensed Matter Field Theory gives the value as



$frac{pi^2}{6} - frac{pi |x|}{2} + frac{x^2}{4} + dots$, giving no domain and with a weird absolute value, implying the series is valid everywhere and has higher powers. But how can this be correct? The series is periodic and those first three terms give the complete answer on $0 leq x leq 2pi$, so trying to define the result everywhere with an infinite power series seems senseless to me.



Anyway, can anybody shed light on how to perform the summation? I've tried writing $S$ as a contour integration, like so:



$frac{1}{2pi i} oint dz g(z) frac{cos(i z x)}{-z^2} $, where $g(z) = frac{beta}{exp(beta z) - 1}$, a counting function with simple poles at $z = i k$, $k = ..., -2, -1, 0, 1, 2, dots$, and the contour contains all the poles of $g(z)$ for $k = 1, 2, 3, ...$.



Now, this is not my expertise, but I want to learn. The trick now is to pick a different contour (possibly going off to infinity), such that the integral can be performed. I see that the product $g(z) frac{cos(i z x)}{-z^2}$ goes to zero at infinity, but I cannot see how to deform the contour such that the integral becomes tractable.







summation residue-calculus






share|cite|improve this question















share|cite|improve this question













share|cite|improve this question




share|cite|improve this question








edited Jan 25 '16 at 10:10









Angelo Mark

4,03721641




4,03721641










asked Jan 25 '16 at 10:08









Kappie001Kappie001

1536




1536








  • 1




    $begingroup$
    "trying to define the result everywhere with an infinite power series seems senseless to me" It does make sense. For instance, $cos(x)$ itself is defined on the entire number line as $$ cos(x) = 1-frac{x^2}2 + frac{x^4}{24} - frac{x^6}{720} + cdots $$As to whether the two solutions you have there coincide, and how the power series is supposed to continue, I do not know.
    $endgroup$
    – Arthur
    Jan 25 '16 at 10:13










  • $begingroup$
    The best is to learn the theory of Bernoulli polynomials. These are periodic function of period $2pi$ whose restriction to the interval $[0,2pi]$ is a polynomial.
    $endgroup$
    – Thomas
    Jan 25 '16 at 10:27










  • $begingroup$
    Mathematica gives $$S=frac{text{Li}_2left(e^{i x}right)+text{Li}_2left(e^{-i x}right)}{2}$$ which isn't really enlightening given what generated it.
    $endgroup$
    – GPhys
    Jan 25 '16 at 10:33












  • $begingroup$
    This is just a Fourier series of a well known function. If you go in reverse, it's quite obvious, just compute the fourier coefficients of $ax^2+bx+c$. How to compute the sum without knowing the answer? I'll find the shortest way and post it below if noone else does.
    $endgroup$
    – orion
    Jan 25 '16 at 11:21














  • 1




    $begingroup$
    "trying to define the result everywhere with an infinite power series seems senseless to me" It does make sense. For instance, $cos(x)$ itself is defined on the entire number line as $$ cos(x) = 1-frac{x^2}2 + frac{x^4}{24} - frac{x^6}{720} + cdots $$As to whether the two solutions you have there coincide, and how the power series is supposed to continue, I do not know.
    $endgroup$
    – Arthur
    Jan 25 '16 at 10:13










  • $begingroup$
    The best is to learn the theory of Bernoulli polynomials. These are periodic function of period $2pi$ whose restriction to the interval $[0,2pi]$ is a polynomial.
    $endgroup$
    – Thomas
    Jan 25 '16 at 10:27










  • $begingroup$
    Mathematica gives $$S=frac{text{Li}_2left(e^{i x}right)+text{Li}_2left(e^{-i x}right)}{2}$$ which isn't really enlightening given what generated it.
    $endgroup$
    – GPhys
    Jan 25 '16 at 10:33












  • $begingroup$
    This is just a Fourier series of a well known function. If you go in reverse, it's quite obvious, just compute the fourier coefficients of $ax^2+bx+c$. How to compute the sum without knowing the answer? I'll find the shortest way and post it below if noone else does.
    $endgroup$
    – orion
    Jan 25 '16 at 11:21








1




1




$begingroup$
"trying to define the result everywhere with an infinite power series seems senseless to me" It does make sense. For instance, $cos(x)$ itself is defined on the entire number line as $$ cos(x) = 1-frac{x^2}2 + frac{x^4}{24} - frac{x^6}{720} + cdots $$As to whether the two solutions you have there coincide, and how the power series is supposed to continue, I do not know.
$endgroup$
– Arthur
Jan 25 '16 at 10:13




$begingroup$
"trying to define the result everywhere with an infinite power series seems senseless to me" It does make sense. For instance, $cos(x)$ itself is defined on the entire number line as $$ cos(x) = 1-frac{x^2}2 + frac{x^4}{24} - frac{x^6}{720} + cdots $$As to whether the two solutions you have there coincide, and how the power series is supposed to continue, I do not know.
$endgroup$
– Arthur
Jan 25 '16 at 10:13












$begingroup$
The best is to learn the theory of Bernoulli polynomials. These are periodic function of period $2pi$ whose restriction to the interval $[0,2pi]$ is a polynomial.
$endgroup$
– Thomas
Jan 25 '16 at 10:27




$begingroup$
The best is to learn the theory of Bernoulli polynomials. These are periodic function of period $2pi$ whose restriction to the interval $[0,2pi]$ is a polynomial.
$endgroup$
– Thomas
Jan 25 '16 at 10:27












$begingroup$
Mathematica gives $$S=frac{text{Li}_2left(e^{i x}right)+text{Li}_2left(e^{-i x}right)}{2}$$ which isn't really enlightening given what generated it.
$endgroup$
– GPhys
Jan 25 '16 at 10:33






$begingroup$
Mathematica gives $$S=frac{text{Li}_2left(e^{i x}right)+text{Li}_2left(e^{-i x}right)}{2}$$ which isn't really enlightening given what generated it.
$endgroup$
– GPhys
Jan 25 '16 at 10:33














$begingroup$
This is just a Fourier series of a well known function. If you go in reverse, it's quite obvious, just compute the fourier coefficients of $ax^2+bx+c$. How to compute the sum without knowing the answer? I'll find the shortest way and post it below if noone else does.
$endgroup$
– orion
Jan 25 '16 at 11:21




$begingroup$
This is just a Fourier series of a well known function. If you go in reverse, it's quite obvious, just compute the fourier coefficients of $ax^2+bx+c$. How to compute the sum without knowing the answer? I'll find the shortest way and post it below if noone else does.
$endgroup$
– orion
Jan 25 '16 at 11:21










3 Answers
3






active

oldest

votes


















4












$begingroup$

It's the real part of this function:



$$f(x)=lim_{epsilon to 0}sum_{k=1}^infty frac{e^{ikx-epsilon k }}{k^2}$$
where $epsilon to 0$. Take two derivatives over $x$, apply expansions you can afford because $epsilon$ is differentially small, compute the sum, integrate back with appropriate boundary conditions (known values for $x=0$).



$$f''(x)=-lim_{epsilon to 0}sum_{k=1}^infty e^{ikx-epsilon k }=lim_{epsilon to 0}frac{e^{ix-epsilon}}{e^{ix-epsilon}-1}$$
$$=lim_{epsilon to 0}frac{e^{ix-epsilon}(e^{-ix-epsilon}-1)}{(e^{ix-epsilon}-1)(e^{-ix-epsilon}-1)}$$
$$=lim_{epsilon to 0}frac{e^{-2epsilon}-e^{-epsilon}e^{ix}}{e^{-2epsilon}-2e^{-epsilon}cos x + 1}=frac{1-e^{ix}}{2(1-cos x)}$$
$$Re f''(x)=frac12$$
$$int int Re f''(x)dx,dx=frac{x^2}{4}+Cx+D$$
We know from Euler's times that $f(0)=frac{pi^2}{6}$, which sets $D=frac{pi^2}{6}$. For $x=pi$ you get another known sum $sum (-1)^k/k^2=-frac{pi^2}{12}$ which gives you $C$.





Disclaimer: we can't integrate past singularities of the $f''(x)$ sum, which is ill-defined when $x$ is a multiple of $2pi$. The integrating constants $C$ and $D$ are different on each subdomain, and the ones above hold between $0$ and $2pi$. The solution is then periodically extended, which is obvious from the properties of $cos$ function. Notice a sharp peak (derivative discontinuity) at $x=0$. If you use an absolute value $|x|$ in the linear term (because you know it must be an even function), you only extend the validity to $[-2pi,2pi]$ interval. Adding extra series terms is misleading - the error between true values and this expression is not polynomial, it's step-function, and it would be quite strange to choose to write periodic steps as a power series.






share|cite|improve this answer











$endgroup$













  • $begingroup$
    Of course the twice differentiated series doesn't converge (the once differentiated series converges if $x$ is not a multiple of $2pi$), so this argument isn't rigorous. One can make it rigorous by introducing a convergence-generating factor $r^k$ ($rin [0,1)$) and taking limits, or by framing it in the context of distributions. As a heuristic argument to find the answer, one need not make it rigorous - but the answer still has to be rigorously justified.
    $endgroup$
    – Daniel Fischer
    Jan 25 '16 at 11:29










  • $begingroup$
    Physicists don't worry about convergence (OP is physicist, and so am I). It works in the sense of summability methods (if you want to be rigorous). All we're doing is extending analytical functions beyond the series convergence (common regularization method).
    $endgroup$
    – orion
    Jan 25 '16 at 11:32










  • $begingroup$
    Additionally, we are in Fourier territory, where almost nothing converges uniformly - it's all in the square integrable sense. Recall the Gibbs phenomenon.
    $endgroup$
    – orion
    Jan 25 '16 at 11:35






  • 1




    $begingroup$
    The twice differentiated series doesn't converge in $L^2$ either, for that you need a more general framework. Nothing wrong with that, but I thought it should at least be mentioned.
    $endgroup$
    – Daniel Fischer
    Jan 25 '16 at 11:44












  • $begingroup$
    Thank you for the reminder. I think now it's ok.
    $endgroup$
    – orion
    Jan 25 '16 at 12:01



















1












$begingroup$

HINT :



Compute the coefficients of the Fourier series for an arc of parabola
$$y(x)=(x-pi)^2 ::::text{for}::::0<x<2pi$$
You will identify $::sum_{k=1}^inftyfrac{cos(kx)}{k^2}::$ in it and then found :
$$sum_{k=1}^inftyfrac{cos(kx)}{k^2}=frac{1}{4}(x-pi)^2-frac{pi^2}{12}$$






share|cite|improve this answer









$endgroup$













  • $begingroup$
    Yes it's easy if you know the answer :) I believe that was one of the earliest ways to compute Euler's sums. But without knowing how to "guess" the function, you have to do it the hard way :) However, it should be noted that among physicists, fourier transforms of polynomials are recognized immediately - the fact that fourier coefficients at $k^{-n}$ signify $x^n$ behaviour of the signal is burned into the minds of everyone that deals with signal processing. So, good to have this answer on record.
    $endgroup$
    – orion
    Jan 25 '16 at 12:26






  • 1




    $begingroup$
    If you don't know the answer, obviously the sum makes you think to Fourier series. Then, considering the integral definition of the coefficients of any Fourier series, what function leads to the coefficient $frac{1}{k^2}$ ? This leads to an integral equation which solving gives the answer : the function is a parabola.
    $endgroup$
    – JJacquelin
    Jan 25 '16 at 12:34










  • $begingroup$
    I know, this was my original hint in the comments, so +1 for this answer, it's the most straightforward one. Just, one can't help but wonder how to solve this without the Ansatz. That's why I also posted a different solution.
    $endgroup$
    – orion
    Jan 25 '16 at 12:41










  • $begingroup$
    @orion : I agree with "the hard way" if one doesn't want to refer to knowledge about Fourier series. So +1 for the hard way.
    $endgroup$
    – JJacquelin
    Jan 25 '16 at 12:58












  • $begingroup$
    What would be $sum_{k=1}^{infty} frac{sin(kx)}{k^2}$ ? Any idea ? Cheers.
    $endgroup$
    – Claude Leibovici
    Jan 27 '16 at 9:36



















1












$begingroup$

This is not a very rigorous answer at all.



Let us consider $$I=sum_{k=1}^{infty} frac{cos(kx)}{k^2}qquad ,qquad J =sum_{k=1}^{infty} frac{sin(kx)}{k^2}$$ $$I+iJ=sum_{k=1}^{infty} frac{e^{ikx}}{k^2}=sum_{k=1}^{infty} frac{(e^{ix})^k}{k^2}=text{Li}_2left(e^{i x}right)$$ where appears the polylogarithm function.



So, $$I=Releft(text{Li}_2left(e^{i x}right)right)=frac{1}{2} left(text{Li}_2left(e^{+i x}right)+text{Li}_2left(e^{-i
x}right)right)$$ $$J=Imleft(text{Li}_2left(e^{i x}right)right)=frac{1}{2} i left(text{Li}_2left(e^{-i x}right)-text{Li}_2left(e^{+i
x}right)right)$$



Since, at this point, being stuck, I used brute force generating a table of $201$ equally spaced values $(0leq x leq 2 pi)$ and performed a quadratic regression which led to $$I=0.25 x^2-1.5708 x+1.64493$$ for a residual sum of squares equal to $1.11times 10^{-27}$ which means a perfect fit.



The second coefficient is obviously $frac pi 2$; concerning the third, if $x=0$ or $x=2pi$, $I=frac{pi^2} 6$. So $$I=frac 14 x^2-frac pi 2 x+frac{pi^2} 6$$ which is the formula from the handbook.



I hope and wish that you will get some more founded answers (as you are, I am a physicist and not an real mathematician).






share|cite|improve this answer











$endgroup$













  • $begingroup$
    This identity can be seemingly shown rigorously using the identity between $mathrm{Li}_2$ and Bernoulli polynomials en.wikipedia.org/wiki/Polylogarithm $mathrm{Li}_n(e^{2pi i x})+(-1)^n mathrm{Li}_n(e^{-2pi i x})=-(2pi i)^n B_n(x)/n!$, so $I=pi^2 B_2(x/(2pi))$.
    $endgroup$
    – Pierpaolo Vivo
    Jan 25 '16 at 11:09












  • $begingroup$
    @PierpaoloVivo. Please make an answer. As I said, I was stuck and then use brute force. Thanks.
    $endgroup$
    – Claude Leibovici
    Jan 25 '16 at 11:15










  • $begingroup$
    @PierpaoloVivo that's great, thanks. I can rest assured now.
    $endgroup$
    – Kappie001
    Jan 25 '16 at 11:35











Your Answer





StackExchange.ifUsing("editor", function () {
return StackExchange.using("mathjaxEditing", function () {
StackExchange.MarkdownEditor.creationCallbacks.add(function (editor, postfix) {
StackExchange.mathjaxEditing.prepareWmdForMathJax(editor, postfix, [["$", "$"], ["\\(","\\)"]]);
});
});
}, "mathjax-editing");

StackExchange.ready(function() {
var channelOptions = {
tags: "".split(" "),
id: "69"
};
initTagRenderer("".split(" "), "".split(" "), channelOptions);

StackExchange.using("externalEditor", function() {
// Have to fire editor after snippets, if snippets enabled
if (StackExchange.settings.snippets.snippetsEnabled) {
StackExchange.using("snippets", function() {
createEditor();
});
}
else {
createEditor();
}
});

function createEditor() {
StackExchange.prepareEditor({
heartbeatType: 'answer',
autoActivateHeartbeat: false,
convertImagesToLinks: true,
noModals: true,
showLowRepImageUploadWarning: true,
reputationToPostImages: 10,
bindNavPrevention: true,
postfix: "",
imageUploader: {
brandingHtml: "Powered by u003ca class="icon-imgur-white" href="https://imgur.com/"u003eu003c/au003e",
contentPolicyHtml: "User contributions licensed under u003ca href="https://creativecommons.org/licenses/by-sa/3.0/"u003ecc by-sa 3.0 with attribution requiredu003c/au003e u003ca href="https://stackoverflow.com/legal/content-policy"u003e(content policy)u003c/au003e",
allowUrls: true
},
noCode: true, onDemand: true,
discardSelector: ".discard-answer"
,immediatelyShowMarkdownHelp:true
});


}
});














draft saved

draft discarded


















StackExchange.ready(
function () {
StackExchange.openid.initPostLogin('.new-post-login', 'https%3a%2f%2fmath.stackexchange.com%2fquestions%2f1626131%2fhow-to-do-sum-k-1-infty-frac-coskxk2%23new-answer', 'question_page');
}
);

Post as a guest















Required, but never shown

























3 Answers
3






active

oldest

votes








3 Answers
3






active

oldest

votes









active

oldest

votes






active

oldest

votes









4












$begingroup$

It's the real part of this function:



$$f(x)=lim_{epsilon to 0}sum_{k=1}^infty frac{e^{ikx-epsilon k }}{k^2}$$
where $epsilon to 0$. Take two derivatives over $x$, apply expansions you can afford because $epsilon$ is differentially small, compute the sum, integrate back with appropriate boundary conditions (known values for $x=0$).



$$f''(x)=-lim_{epsilon to 0}sum_{k=1}^infty e^{ikx-epsilon k }=lim_{epsilon to 0}frac{e^{ix-epsilon}}{e^{ix-epsilon}-1}$$
$$=lim_{epsilon to 0}frac{e^{ix-epsilon}(e^{-ix-epsilon}-1)}{(e^{ix-epsilon}-1)(e^{-ix-epsilon}-1)}$$
$$=lim_{epsilon to 0}frac{e^{-2epsilon}-e^{-epsilon}e^{ix}}{e^{-2epsilon}-2e^{-epsilon}cos x + 1}=frac{1-e^{ix}}{2(1-cos x)}$$
$$Re f''(x)=frac12$$
$$int int Re f''(x)dx,dx=frac{x^2}{4}+Cx+D$$
We know from Euler's times that $f(0)=frac{pi^2}{6}$, which sets $D=frac{pi^2}{6}$. For $x=pi$ you get another known sum $sum (-1)^k/k^2=-frac{pi^2}{12}$ which gives you $C$.





Disclaimer: we can't integrate past singularities of the $f''(x)$ sum, which is ill-defined when $x$ is a multiple of $2pi$. The integrating constants $C$ and $D$ are different on each subdomain, and the ones above hold between $0$ and $2pi$. The solution is then periodically extended, which is obvious from the properties of $cos$ function. Notice a sharp peak (derivative discontinuity) at $x=0$. If you use an absolute value $|x|$ in the linear term (because you know it must be an even function), you only extend the validity to $[-2pi,2pi]$ interval. Adding extra series terms is misleading - the error between true values and this expression is not polynomial, it's step-function, and it would be quite strange to choose to write periodic steps as a power series.






share|cite|improve this answer











$endgroup$













  • $begingroup$
    Of course the twice differentiated series doesn't converge (the once differentiated series converges if $x$ is not a multiple of $2pi$), so this argument isn't rigorous. One can make it rigorous by introducing a convergence-generating factor $r^k$ ($rin [0,1)$) and taking limits, or by framing it in the context of distributions. As a heuristic argument to find the answer, one need not make it rigorous - but the answer still has to be rigorously justified.
    $endgroup$
    – Daniel Fischer
    Jan 25 '16 at 11:29










  • $begingroup$
    Physicists don't worry about convergence (OP is physicist, and so am I). It works in the sense of summability methods (if you want to be rigorous). All we're doing is extending analytical functions beyond the series convergence (common regularization method).
    $endgroup$
    – orion
    Jan 25 '16 at 11:32










  • $begingroup$
    Additionally, we are in Fourier territory, where almost nothing converges uniformly - it's all in the square integrable sense. Recall the Gibbs phenomenon.
    $endgroup$
    – orion
    Jan 25 '16 at 11:35






  • 1




    $begingroup$
    The twice differentiated series doesn't converge in $L^2$ either, for that you need a more general framework. Nothing wrong with that, but I thought it should at least be mentioned.
    $endgroup$
    – Daniel Fischer
    Jan 25 '16 at 11:44












  • $begingroup$
    Thank you for the reminder. I think now it's ok.
    $endgroup$
    – orion
    Jan 25 '16 at 12:01
















4












$begingroup$

It's the real part of this function:



$$f(x)=lim_{epsilon to 0}sum_{k=1}^infty frac{e^{ikx-epsilon k }}{k^2}$$
where $epsilon to 0$. Take two derivatives over $x$, apply expansions you can afford because $epsilon$ is differentially small, compute the sum, integrate back with appropriate boundary conditions (known values for $x=0$).



$$f''(x)=-lim_{epsilon to 0}sum_{k=1}^infty e^{ikx-epsilon k }=lim_{epsilon to 0}frac{e^{ix-epsilon}}{e^{ix-epsilon}-1}$$
$$=lim_{epsilon to 0}frac{e^{ix-epsilon}(e^{-ix-epsilon}-1)}{(e^{ix-epsilon}-1)(e^{-ix-epsilon}-1)}$$
$$=lim_{epsilon to 0}frac{e^{-2epsilon}-e^{-epsilon}e^{ix}}{e^{-2epsilon}-2e^{-epsilon}cos x + 1}=frac{1-e^{ix}}{2(1-cos x)}$$
$$Re f''(x)=frac12$$
$$int int Re f''(x)dx,dx=frac{x^2}{4}+Cx+D$$
We know from Euler's times that $f(0)=frac{pi^2}{6}$, which sets $D=frac{pi^2}{6}$. For $x=pi$ you get another known sum $sum (-1)^k/k^2=-frac{pi^2}{12}$ which gives you $C$.





Disclaimer: we can't integrate past singularities of the $f''(x)$ sum, which is ill-defined when $x$ is a multiple of $2pi$. The integrating constants $C$ and $D$ are different on each subdomain, and the ones above hold between $0$ and $2pi$. The solution is then periodically extended, which is obvious from the properties of $cos$ function. Notice a sharp peak (derivative discontinuity) at $x=0$. If you use an absolute value $|x|$ in the linear term (because you know it must be an even function), you only extend the validity to $[-2pi,2pi]$ interval. Adding extra series terms is misleading - the error between true values and this expression is not polynomial, it's step-function, and it would be quite strange to choose to write periodic steps as a power series.






share|cite|improve this answer











$endgroup$













  • $begingroup$
    Of course the twice differentiated series doesn't converge (the once differentiated series converges if $x$ is not a multiple of $2pi$), so this argument isn't rigorous. One can make it rigorous by introducing a convergence-generating factor $r^k$ ($rin [0,1)$) and taking limits, or by framing it in the context of distributions. As a heuristic argument to find the answer, one need not make it rigorous - but the answer still has to be rigorously justified.
    $endgroup$
    – Daniel Fischer
    Jan 25 '16 at 11:29










  • $begingroup$
    Physicists don't worry about convergence (OP is physicist, and so am I). It works in the sense of summability methods (if you want to be rigorous). All we're doing is extending analytical functions beyond the series convergence (common regularization method).
    $endgroup$
    – orion
    Jan 25 '16 at 11:32










  • $begingroup$
    Additionally, we are in Fourier territory, where almost nothing converges uniformly - it's all in the square integrable sense. Recall the Gibbs phenomenon.
    $endgroup$
    – orion
    Jan 25 '16 at 11:35






  • 1




    $begingroup$
    The twice differentiated series doesn't converge in $L^2$ either, for that you need a more general framework. Nothing wrong with that, but I thought it should at least be mentioned.
    $endgroup$
    – Daniel Fischer
    Jan 25 '16 at 11:44












  • $begingroup$
    Thank you for the reminder. I think now it's ok.
    $endgroup$
    – orion
    Jan 25 '16 at 12:01














4












4








4





$begingroup$

It's the real part of this function:



$$f(x)=lim_{epsilon to 0}sum_{k=1}^infty frac{e^{ikx-epsilon k }}{k^2}$$
where $epsilon to 0$. Take two derivatives over $x$, apply expansions you can afford because $epsilon$ is differentially small, compute the sum, integrate back with appropriate boundary conditions (known values for $x=0$).



$$f''(x)=-lim_{epsilon to 0}sum_{k=1}^infty e^{ikx-epsilon k }=lim_{epsilon to 0}frac{e^{ix-epsilon}}{e^{ix-epsilon}-1}$$
$$=lim_{epsilon to 0}frac{e^{ix-epsilon}(e^{-ix-epsilon}-1)}{(e^{ix-epsilon}-1)(e^{-ix-epsilon}-1)}$$
$$=lim_{epsilon to 0}frac{e^{-2epsilon}-e^{-epsilon}e^{ix}}{e^{-2epsilon}-2e^{-epsilon}cos x + 1}=frac{1-e^{ix}}{2(1-cos x)}$$
$$Re f''(x)=frac12$$
$$int int Re f''(x)dx,dx=frac{x^2}{4}+Cx+D$$
We know from Euler's times that $f(0)=frac{pi^2}{6}$, which sets $D=frac{pi^2}{6}$. For $x=pi$ you get another known sum $sum (-1)^k/k^2=-frac{pi^2}{12}$ which gives you $C$.





Disclaimer: we can't integrate past singularities of the $f''(x)$ sum, which is ill-defined when $x$ is a multiple of $2pi$. The integrating constants $C$ and $D$ are different on each subdomain, and the ones above hold between $0$ and $2pi$. The solution is then periodically extended, which is obvious from the properties of $cos$ function. Notice a sharp peak (derivative discontinuity) at $x=0$. If you use an absolute value $|x|$ in the linear term (because you know it must be an even function), you only extend the validity to $[-2pi,2pi]$ interval. Adding extra series terms is misleading - the error between true values and this expression is not polynomial, it's step-function, and it would be quite strange to choose to write periodic steps as a power series.






share|cite|improve this answer











$endgroup$



It's the real part of this function:



$$f(x)=lim_{epsilon to 0}sum_{k=1}^infty frac{e^{ikx-epsilon k }}{k^2}$$
where $epsilon to 0$. Take two derivatives over $x$, apply expansions you can afford because $epsilon$ is differentially small, compute the sum, integrate back with appropriate boundary conditions (known values for $x=0$).



$$f''(x)=-lim_{epsilon to 0}sum_{k=1}^infty e^{ikx-epsilon k }=lim_{epsilon to 0}frac{e^{ix-epsilon}}{e^{ix-epsilon}-1}$$
$$=lim_{epsilon to 0}frac{e^{ix-epsilon}(e^{-ix-epsilon}-1)}{(e^{ix-epsilon}-1)(e^{-ix-epsilon}-1)}$$
$$=lim_{epsilon to 0}frac{e^{-2epsilon}-e^{-epsilon}e^{ix}}{e^{-2epsilon}-2e^{-epsilon}cos x + 1}=frac{1-e^{ix}}{2(1-cos x)}$$
$$Re f''(x)=frac12$$
$$int int Re f''(x)dx,dx=frac{x^2}{4}+Cx+D$$
We know from Euler's times that $f(0)=frac{pi^2}{6}$, which sets $D=frac{pi^2}{6}$. For $x=pi$ you get another known sum $sum (-1)^k/k^2=-frac{pi^2}{12}$ which gives you $C$.





Disclaimer: we can't integrate past singularities of the $f''(x)$ sum, which is ill-defined when $x$ is a multiple of $2pi$. The integrating constants $C$ and $D$ are different on each subdomain, and the ones above hold between $0$ and $2pi$. The solution is then periodically extended, which is obvious from the properties of $cos$ function. Notice a sharp peak (derivative discontinuity) at $x=0$. If you use an absolute value $|x|$ in the linear term (because you know it must be an even function), you only extend the validity to $[-2pi,2pi]$ interval. Adding extra series terms is misleading - the error between true values and this expression is not polynomial, it's step-function, and it would be quite strange to choose to write periodic steps as a power series.







share|cite|improve this answer














share|cite|improve this answer



share|cite|improve this answer








edited Jan 17 at 10:13

























answered Jan 25 '16 at 11:22









orionorion

13.6k11837




13.6k11837












  • $begingroup$
    Of course the twice differentiated series doesn't converge (the once differentiated series converges if $x$ is not a multiple of $2pi$), so this argument isn't rigorous. One can make it rigorous by introducing a convergence-generating factor $r^k$ ($rin [0,1)$) and taking limits, or by framing it in the context of distributions. As a heuristic argument to find the answer, one need not make it rigorous - but the answer still has to be rigorously justified.
    $endgroup$
    – Daniel Fischer
    Jan 25 '16 at 11:29










  • $begingroup$
    Physicists don't worry about convergence (OP is physicist, and so am I). It works in the sense of summability methods (if you want to be rigorous). All we're doing is extending analytical functions beyond the series convergence (common regularization method).
    $endgroup$
    – orion
    Jan 25 '16 at 11:32










  • $begingroup$
    Additionally, we are in Fourier territory, where almost nothing converges uniformly - it's all in the square integrable sense. Recall the Gibbs phenomenon.
    $endgroup$
    – orion
    Jan 25 '16 at 11:35






  • 1




    $begingroup$
    The twice differentiated series doesn't converge in $L^2$ either, for that you need a more general framework. Nothing wrong with that, but I thought it should at least be mentioned.
    $endgroup$
    – Daniel Fischer
    Jan 25 '16 at 11:44












  • $begingroup$
    Thank you for the reminder. I think now it's ok.
    $endgroup$
    – orion
    Jan 25 '16 at 12:01


















  • $begingroup$
    Of course the twice differentiated series doesn't converge (the once differentiated series converges if $x$ is not a multiple of $2pi$), so this argument isn't rigorous. One can make it rigorous by introducing a convergence-generating factor $r^k$ ($rin [0,1)$) and taking limits, or by framing it in the context of distributions. As a heuristic argument to find the answer, one need not make it rigorous - but the answer still has to be rigorously justified.
    $endgroup$
    – Daniel Fischer
    Jan 25 '16 at 11:29










  • $begingroup$
    Physicists don't worry about convergence (OP is physicist, and so am I). It works in the sense of summability methods (if you want to be rigorous). All we're doing is extending analytical functions beyond the series convergence (common regularization method).
    $endgroup$
    – orion
    Jan 25 '16 at 11:32










  • $begingroup$
    Additionally, we are in Fourier territory, where almost nothing converges uniformly - it's all in the square integrable sense. Recall the Gibbs phenomenon.
    $endgroup$
    – orion
    Jan 25 '16 at 11:35






  • 1




    $begingroup$
    The twice differentiated series doesn't converge in $L^2$ either, for that you need a more general framework. Nothing wrong with that, but I thought it should at least be mentioned.
    $endgroup$
    – Daniel Fischer
    Jan 25 '16 at 11:44












  • $begingroup$
    Thank you for the reminder. I think now it's ok.
    $endgroup$
    – orion
    Jan 25 '16 at 12:01
















$begingroup$
Of course the twice differentiated series doesn't converge (the once differentiated series converges if $x$ is not a multiple of $2pi$), so this argument isn't rigorous. One can make it rigorous by introducing a convergence-generating factor $r^k$ ($rin [0,1)$) and taking limits, or by framing it in the context of distributions. As a heuristic argument to find the answer, one need not make it rigorous - but the answer still has to be rigorously justified.
$endgroup$
– Daniel Fischer
Jan 25 '16 at 11:29




$begingroup$
Of course the twice differentiated series doesn't converge (the once differentiated series converges if $x$ is not a multiple of $2pi$), so this argument isn't rigorous. One can make it rigorous by introducing a convergence-generating factor $r^k$ ($rin [0,1)$) and taking limits, or by framing it in the context of distributions. As a heuristic argument to find the answer, one need not make it rigorous - but the answer still has to be rigorously justified.
$endgroup$
– Daniel Fischer
Jan 25 '16 at 11:29












$begingroup$
Physicists don't worry about convergence (OP is physicist, and so am I). It works in the sense of summability methods (if you want to be rigorous). All we're doing is extending analytical functions beyond the series convergence (common regularization method).
$endgroup$
– orion
Jan 25 '16 at 11:32




$begingroup$
Physicists don't worry about convergence (OP is physicist, and so am I). It works in the sense of summability methods (if you want to be rigorous). All we're doing is extending analytical functions beyond the series convergence (common regularization method).
$endgroup$
– orion
Jan 25 '16 at 11:32












$begingroup$
Additionally, we are in Fourier territory, where almost nothing converges uniformly - it's all in the square integrable sense. Recall the Gibbs phenomenon.
$endgroup$
– orion
Jan 25 '16 at 11:35




$begingroup$
Additionally, we are in Fourier territory, where almost nothing converges uniformly - it's all in the square integrable sense. Recall the Gibbs phenomenon.
$endgroup$
– orion
Jan 25 '16 at 11:35




1




1




$begingroup$
The twice differentiated series doesn't converge in $L^2$ either, for that you need a more general framework. Nothing wrong with that, but I thought it should at least be mentioned.
$endgroup$
– Daniel Fischer
Jan 25 '16 at 11:44






$begingroup$
The twice differentiated series doesn't converge in $L^2$ either, for that you need a more general framework. Nothing wrong with that, but I thought it should at least be mentioned.
$endgroup$
– Daniel Fischer
Jan 25 '16 at 11:44














$begingroup$
Thank you for the reminder. I think now it's ok.
$endgroup$
– orion
Jan 25 '16 at 12:01




$begingroup$
Thank you for the reminder. I think now it's ok.
$endgroup$
– orion
Jan 25 '16 at 12:01











1












$begingroup$

HINT :



Compute the coefficients of the Fourier series for an arc of parabola
$$y(x)=(x-pi)^2 ::::text{for}::::0<x<2pi$$
You will identify $::sum_{k=1}^inftyfrac{cos(kx)}{k^2}::$ in it and then found :
$$sum_{k=1}^inftyfrac{cos(kx)}{k^2}=frac{1}{4}(x-pi)^2-frac{pi^2}{12}$$






share|cite|improve this answer









$endgroup$













  • $begingroup$
    Yes it's easy if you know the answer :) I believe that was one of the earliest ways to compute Euler's sums. But without knowing how to "guess" the function, you have to do it the hard way :) However, it should be noted that among physicists, fourier transforms of polynomials are recognized immediately - the fact that fourier coefficients at $k^{-n}$ signify $x^n$ behaviour of the signal is burned into the minds of everyone that deals with signal processing. So, good to have this answer on record.
    $endgroup$
    – orion
    Jan 25 '16 at 12:26






  • 1




    $begingroup$
    If you don't know the answer, obviously the sum makes you think to Fourier series. Then, considering the integral definition of the coefficients of any Fourier series, what function leads to the coefficient $frac{1}{k^2}$ ? This leads to an integral equation which solving gives the answer : the function is a parabola.
    $endgroup$
    – JJacquelin
    Jan 25 '16 at 12:34










  • $begingroup$
    I know, this was my original hint in the comments, so +1 for this answer, it's the most straightforward one. Just, one can't help but wonder how to solve this without the Ansatz. That's why I also posted a different solution.
    $endgroup$
    – orion
    Jan 25 '16 at 12:41










  • $begingroup$
    @orion : I agree with "the hard way" if one doesn't want to refer to knowledge about Fourier series. So +1 for the hard way.
    $endgroup$
    – JJacquelin
    Jan 25 '16 at 12:58












  • $begingroup$
    What would be $sum_{k=1}^{infty} frac{sin(kx)}{k^2}$ ? Any idea ? Cheers.
    $endgroup$
    – Claude Leibovici
    Jan 27 '16 at 9:36
















1












$begingroup$

HINT :



Compute the coefficients of the Fourier series for an arc of parabola
$$y(x)=(x-pi)^2 ::::text{for}::::0<x<2pi$$
You will identify $::sum_{k=1}^inftyfrac{cos(kx)}{k^2}::$ in it and then found :
$$sum_{k=1}^inftyfrac{cos(kx)}{k^2}=frac{1}{4}(x-pi)^2-frac{pi^2}{12}$$






share|cite|improve this answer









$endgroup$













  • $begingroup$
    Yes it's easy if you know the answer :) I believe that was one of the earliest ways to compute Euler's sums. But without knowing how to "guess" the function, you have to do it the hard way :) However, it should be noted that among physicists, fourier transforms of polynomials are recognized immediately - the fact that fourier coefficients at $k^{-n}$ signify $x^n$ behaviour of the signal is burned into the minds of everyone that deals with signal processing. So, good to have this answer on record.
    $endgroup$
    – orion
    Jan 25 '16 at 12:26






  • 1




    $begingroup$
    If you don't know the answer, obviously the sum makes you think to Fourier series. Then, considering the integral definition of the coefficients of any Fourier series, what function leads to the coefficient $frac{1}{k^2}$ ? This leads to an integral equation which solving gives the answer : the function is a parabola.
    $endgroup$
    – JJacquelin
    Jan 25 '16 at 12:34










  • $begingroup$
    I know, this was my original hint in the comments, so +1 for this answer, it's the most straightforward one. Just, one can't help but wonder how to solve this without the Ansatz. That's why I also posted a different solution.
    $endgroup$
    – orion
    Jan 25 '16 at 12:41










  • $begingroup$
    @orion : I agree with "the hard way" if one doesn't want to refer to knowledge about Fourier series. So +1 for the hard way.
    $endgroup$
    – JJacquelin
    Jan 25 '16 at 12:58












  • $begingroup$
    What would be $sum_{k=1}^{infty} frac{sin(kx)}{k^2}$ ? Any idea ? Cheers.
    $endgroup$
    – Claude Leibovici
    Jan 27 '16 at 9:36














1












1








1





$begingroup$

HINT :



Compute the coefficients of the Fourier series for an arc of parabola
$$y(x)=(x-pi)^2 ::::text{for}::::0<x<2pi$$
You will identify $::sum_{k=1}^inftyfrac{cos(kx)}{k^2}::$ in it and then found :
$$sum_{k=1}^inftyfrac{cos(kx)}{k^2}=frac{1}{4}(x-pi)^2-frac{pi^2}{12}$$






share|cite|improve this answer









$endgroup$



HINT :



Compute the coefficients of the Fourier series for an arc of parabola
$$y(x)=(x-pi)^2 ::::text{for}::::0<x<2pi$$
You will identify $::sum_{k=1}^inftyfrac{cos(kx)}{k^2}::$ in it and then found :
$$sum_{k=1}^inftyfrac{cos(kx)}{k^2}=frac{1}{4}(x-pi)^2-frac{pi^2}{12}$$







share|cite|improve this answer












share|cite|improve this answer



share|cite|improve this answer










answered Jan 25 '16 at 12:18









JJacquelinJJacquelin

43.7k21853




43.7k21853












  • $begingroup$
    Yes it's easy if you know the answer :) I believe that was one of the earliest ways to compute Euler's sums. But without knowing how to "guess" the function, you have to do it the hard way :) However, it should be noted that among physicists, fourier transforms of polynomials are recognized immediately - the fact that fourier coefficients at $k^{-n}$ signify $x^n$ behaviour of the signal is burned into the minds of everyone that deals with signal processing. So, good to have this answer on record.
    $endgroup$
    – orion
    Jan 25 '16 at 12:26






  • 1




    $begingroup$
    If you don't know the answer, obviously the sum makes you think to Fourier series. Then, considering the integral definition of the coefficients of any Fourier series, what function leads to the coefficient $frac{1}{k^2}$ ? This leads to an integral equation which solving gives the answer : the function is a parabola.
    $endgroup$
    – JJacquelin
    Jan 25 '16 at 12:34










  • $begingroup$
    I know, this was my original hint in the comments, so +1 for this answer, it's the most straightforward one. Just, one can't help but wonder how to solve this without the Ansatz. That's why I also posted a different solution.
    $endgroup$
    – orion
    Jan 25 '16 at 12:41










  • $begingroup$
    @orion : I agree with "the hard way" if one doesn't want to refer to knowledge about Fourier series. So +1 for the hard way.
    $endgroup$
    – JJacquelin
    Jan 25 '16 at 12:58












  • $begingroup$
    What would be $sum_{k=1}^{infty} frac{sin(kx)}{k^2}$ ? Any idea ? Cheers.
    $endgroup$
    – Claude Leibovici
    Jan 27 '16 at 9:36


















  • $begingroup$
    Yes it's easy if you know the answer :) I believe that was one of the earliest ways to compute Euler's sums. But without knowing how to "guess" the function, you have to do it the hard way :) However, it should be noted that among physicists, fourier transforms of polynomials are recognized immediately - the fact that fourier coefficients at $k^{-n}$ signify $x^n$ behaviour of the signal is burned into the minds of everyone that deals with signal processing. So, good to have this answer on record.
    $endgroup$
    – orion
    Jan 25 '16 at 12:26






  • 1




    $begingroup$
    If you don't know the answer, obviously the sum makes you think to Fourier series. Then, considering the integral definition of the coefficients of any Fourier series, what function leads to the coefficient $frac{1}{k^2}$ ? This leads to an integral equation which solving gives the answer : the function is a parabola.
    $endgroup$
    – JJacquelin
    Jan 25 '16 at 12:34










  • $begingroup$
    I know, this was my original hint in the comments, so +1 for this answer, it's the most straightforward one. Just, one can't help but wonder how to solve this without the Ansatz. That's why I also posted a different solution.
    $endgroup$
    – orion
    Jan 25 '16 at 12:41










  • $begingroup$
    @orion : I agree with "the hard way" if one doesn't want to refer to knowledge about Fourier series. So +1 for the hard way.
    $endgroup$
    – JJacquelin
    Jan 25 '16 at 12:58












  • $begingroup$
    What would be $sum_{k=1}^{infty} frac{sin(kx)}{k^2}$ ? Any idea ? Cheers.
    $endgroup$
    – Claude Leibovici
    Jan 27 '16 at 9:36
















$begingroup$
Yes it's easy if you know the answer :) I believe that was one of the earliest ways to compute Euler's sums. But without knowing how to "guess" the function, you have to do it the hard way :) However, it should be noted that among physicists, fourier transforms of polynomials are recognized immediately - the fact that fourier coefficients at $k^{-n}$ signify $x^n$ behaviour of the signal is burned into the minds of everyone that deals with signal processing. So, good to have this answer on record.
$endgroup$
– orion
Jan 25 '16 at 12:26




$begingroup$
Yes it's easy if you know the answer :) I believe that was one of the earliest ways to compute Euler's sums. But without knowing how to "guess" the function, you have to do it the hard way :) However, it should be noted that among physicists, fourier transforms of polynomials are recognized immediately - the fact that fourier coefficients at $k^{-n}$ signify $x^n$ behaviour of the signal is burned into the minds of everyone that deals with signal processing. So, good to have this answer on record.
$endgroup$
– orion
Jan 25 '16 at 12:26




1




1




$begingroup$
If you don't know the answer, obviously the sum makes you think to Fourier series. Then, considering the integral definition of the coefficients of any Fourier series, what function leads to the coefficient $frac{1}{k^2}$ ? This leads to an integral equation which solving gives the answer : the function is a parabola.
$endgroup$
– JJacquelin
Jan 25 '16 at 12:34




$begingroup$
If you don't know the answer, obviously the sum makes you think to Fourier series. Then, considering the integral definition of the coefficients of any Fourier series, what function leads to the coefficient $frac{1}{k^2}$ ? This leads to an integral equation which solving gives the answer : the function is a parabola.
$endgroup$
– JJacquelin
Jan 25 '16 at 12:34












$begingroup$
I know, this was my original hint in the comments, so +1 for this answer, it's the most straightforward one. Just, one can't help but wonder how to solve this without the Ansatz. That's why I also posted a different solution.
$endgroup$
– orion
Jan 25 '16 at 12:41




$begingroup$
I know, this was my original hint in the comments, so +1 for this answer, it's the most straightforward one. Just, one can't help but wonder how to solve this without the Ansatz. That's why I also posted a different solution.
$endgroup$
– orion
Jan 25 '16 at 12:41












$begingroup$
@orion : I agree with "the hard way" if one doesn't want to refer to knowledge about Fourier series. So +1 for the hard way.
$endgroup$
– JJacquelin
Jan 25 '16 at 12:58






$begingroup$
@orion : I agree with "the hard way" if one doesn't want to refer to knowledge about Fourier series. So +1 for the hard way.
$endgroup$
– JJacquelin
Jan 25 '16 at 12:58














$begingroup$
What would be $sum_{k=1}^{infty} frac{sin(kx)}{k^2}$ ? Any idea ? Cheers.
$endgroup$
– Claude Leibovici
Jan 27 '16 at 9:36




$begingroup$
What would be $sum_{k=1}^{infty} frac{sin(kx)}{k^2}$ ? Any idea ? Cheers.
$endgroup$
– Claude Leibovici
Jan 27 '16 at 9:36











1












$begingroup$

This is not a very rigorous answer at all.



Let us consider $$I=sum_{k=1}^{infty} frac{cos(kx)}{k^2}qquad ,qquad J =sum_{k=1}^{infty} frac{sin(kx)}{k^2}$$ $$I+iJ=sum_{k=1}^{infty} frac{e^{ikx}}{k^2}=sum_{k=1}^{infty} frac{(e^{ix})^k}{k^2}=text{Li}_2left(e^{i x}right)$$ where appears the polylogarithm function.



So, $$I=Releft(text{Li}_2left(e^{i x}right)right)=frac{1}{2} left(text{Li}_2left(e^{+i x}right)+text{Li}_2left(e^{-i
x}right)right)$$ $$J=Imleft(text{Li}_2left(e^{i x}right)right)=frac{1}{2} i left(text{Li}_2left(e^{-i x}right)-text{Li}_2left(e^{+i
x}right)right)$$



Since, at this point, being stuck, I used brute force generating a table of $201$ equally spaced values $(0leq x leq 2 pi)$ and performed a quadratic regression which led to $$I=0.25 x^2-1.5708 x+1.64493$$ for a residual sum of squares equal to $1.11times 10^{-27}$ which means a perfect fit.



The second coefficient is obviously $frac pi 2$; concerning the third, if $x=0$ or $x=2pi$, $I=frac{pi^2} 6$. So $$I=frac 14 x^2-frac pi 2 x+frac{pi^2} 6$$ which is the formula from the handbook.



I hope and wish that you will get some more founded answers (as you are, I am a physicist and not an real mathematician).






share|cite|improve this answer











$endgroup$













  • $begingroup$
    This identity can be seemingly shown rigorously using the identity between $mathrm{Li}_2$ and Bernoulli polynomials en.wikipedia.org/wiki/Polylogarithm $mathrm{Li}_n(e^{2pi i x})+(-1)^n mathrm{Li}_n(e^{-2pi i x})=-(2pi i)^n B_n(x)/n!$, so $I=pi^2 B_2(x/(2pi))$.
    $endgroup$
    – Pierpaolo Vivo
    Jan 25 '16 at 11:09












  • $begingroup$
    @PierpaoloVivo. Please make an answer. As I said, I was stuck and then use brute force. Thanks.
    $endgroup$
    – Claude Leibovici
    Jan 25 '16 at 11:15










  • $begingroup$
    @PierpaoloVivo that's great, thanks. I can rest assured now.
    $endgroup$
    – Kappie001
    Jan 25 '16 at 11:35
















1












$begingroup$

This is not a very rigorous answer at all.



Let us consider $$I=sum_{k=1}^{infty} frac{cos(kx)}{k^2}qquad ,qquad J =sum_{k=1}^{infty} frac{sin(kx)}{k^2}$$ $$I+iJ=sum_{k=1}^{infty} frac{e^{ikx}}{k^2}=sum_{k=1}^{infty} frac{(e^{ix})^k}{k^2}=text{Li}_2left(e^{i x}right)$$ where appears the polylogarithm function.



So, $$I=Releft(text{Li}_2left(e^{i x}right)right)=frac{1}{2} left(text{Li}_2left(e^{+i x}right)+text{Li}_2left(e^{-i
x}right)right)$$ $$J=Imleft(text{Li}_2left(e^{i x}right)right)=frac{1}{2} i left(text{Li}_2left(e^{-i x}right)-text{Li}_2left(e^{+i
x}right)right)$$



Since, at this point, being stuck, I used brute force generating a table of $201$ equally spaced values $(0leq x leq 2 pi)$ and performed a quadratic regression which led to $$I=0.25 x^2-1.5708 x+1.64493$$ for a residual sum of squares equal to $1.11times 10^{-27}$ which means a perfect fit.



The second coefficient is obviously $frac pi 2$; concerning the third, if $x=0$ or $x=2pi$, $I=frac{pi^2} 6$. So $$I=frac 14 x^2-frac pi 2 x+frac{pi^2} 6$$ which is the formula from the handbook.



I hope and wish that you will get some more founded answers (as you are, I am a physicist and not an real mathematician).






share|cite|improve this answer











$endgroup$













  • $begingroup$
    This identity can be seemingly shown rigorously using the identity between $mathrm{Li}_2$ and Bernoulli polynomials en.wikipedia.org/wiki/Polylogarithm $mathrm{Li}_n(e^{2pi i x})+(-1)^n mathrm{Li}_n(e^{-2pi i x})=-(2pi i)^n B_n(x)/n!$, so $I=pi^2 B_2(x/(2pi))$.
    $endgroup$
    – Pierpaolo Vivo
    Jan 25 '16 at 11:09












  • $begingroup$
    @PierpaoloVivo. Please make an answer. As I said, I was stuck and then use brute force. Thanks.
    $endgroup$
    – Claude Leibovici
    Jan 25 '16 at 11:15










  • $begingroup$
    @PierpaoloVivo that's great, thanks. I can rest assured now.
    $endgroup$
    – Kappie001
    Jan 25 '16 at 11:35














1












1








1





$begingroup$

This is not a very rigorous answer at all.



Let us consider $$I=sum_{k=1}^{infty} frac{cos(kx)}{k^2}qquad ,qquad J =sum_{k=1}^{infty} frac{sin(kx)}{k^2}$$ $$I+iJ=sum_{k=1}^{infty} frac{e^{ikx}}{k^2}=sum_{k=1}^{infty} frac{(e^{ix})^k}{k^2}=text{Li}_2left(e^{i x}right)$$ where appears the polylogarithm function.



So, $$I=Releft(text{Li}_2left(e^{i x}right)right)=frac{1}{2} left(text{Li}_2left(e^{+i x}right)+text{Li}_2left(e^{-i
x}right)right)$$ $$J=Imleft(text{Li}_2left(e^{i x}right)right)=frac{1}{2} i left(text{Li}_2left(e^{-i x}right)-text{Li}_2left(e^{+i
x}right)right)$$



Since, at this point, being stuck, I used brute force generating a table of $201$ equally spaced values $(0leq x leq 2 pi)$ and performed a quadratic regression which led to $$I=0.25 x^2-1.5708 x+1.64493$$ for a residual sum of squares equal to $1.11times 10^{-27}$ which means a perfect fit.



The second coefficient is obviously $frac pi 2$; concerning the third, if $x=0$ or $x=2pi$, $I=frac{pi^2} 6$. So $$I=frac 14 x^2-frac pi 2 x+frac{pi^2} 6$$ which is the formula from the handbook.



I hope and wish that you will get some more founded answers (as you are, I am a physicist and not an real mathematician).






share|cite|improve this answer











$endgroup$



This is not a very rigorous answer at all.



Let us consider $$I=sum_{k=1}^{infty} frac{cos(kx)}{k^2}qquad ,qquad J =sum_{k=1}^{infty} frac{sin(kx)}{k^2}$$ $$I+iJ=sum_{k=1}^{infty} frac{e^{ikx}}{k^2}=sum_{k=1}^{infty} frac{(e^{ix})^k}{k^2}=text{Li}_2left(e^{i x}right)$$ where appears the polylogarithm function.



So, $$I=Releft(text{Li}_2left(e^{i x}right)right)=frac{1}{2} left(text{Li}_2left(e^{+i x}right)+text{Li}_2left(e^{-i
x}right)right)$$ $$J=Imleft(text{Li}_2left(e^{i x}right)right)=frac{1}{2} i left(text{Li}_2left(e^{-i x}right)-text{Li}_2left(e^{+i
x}right)right)$$



Since, at this point, being stuck, I used brute force generating a table of $201$ equally spaced values $(0leq x leq 2 pi)$ and performed a quadratic regression which led to $$I=0.25 x^2-1.5708 x+1.64493$$ for a residual sum of squares equal to $1.11times 10^{-27}$ which means a perfect fit.



The second coefficient is obviously $frac pi 2$; concerning the third, if $x=0$ or $x=2pi$, $I=frac{pi^2} 6$. So $$I=frac 14 x^2-frac pi 2 x+frac{pi^2} 6$$ which is the formula from the handbook.



I hope and wish that you will get some more founded answers (as you are, I am a physicist and not an real mathematician).







share|cite|improve this answer














share|cite|improve this answer



share|cite|improve this answer








edited Jan 26 '16 at 3:19

























answered Jan 25 '16 at 10:54









Claude LeiboviciClaude Leibovici

121k1157133




121k1157133












  • $begingroup$
    This identity can be seemingly shown rigorously using the identity between $mathrm{Li}_2$ and Bernoulli polynomials en.wikipedia.org/wiki/Polylogarithm $mathrm{Li}_n(e^{2pi i x})+(-1)^n mathrm{Li}_n(e^{-2pi i x})=-(2pi i)^n B_n(x)/n!$, so $I=pi^2 B_2(x/(2pi))$.
    $endgroup$
    – Pierpaolo Vivo
    Jan 25 '16 at 11:09












  • $begingroup$
    @PierpaoloVivo. Please make an answer. As I said, I was stuck and then use brute force. Thanks.
    $endgroup$
    – Claude Leibovici
    Jan 25 '16 at 11:15










  • $begingroup$
    @PierpaoloVivo that's great, thanks. I can rest assured now.
    $endgroup$
    – Kappie001
    Jan 25 '16 at 11:35


















  • $begingroup$
    This identity can be seemingly shown rigorously using the identity between $mathrm{Li}_2$ and Bernoulli polynomials en.wikipedia.org/wiki/Polylogarithm $mathrm{Li}_n(e^{2pi i x})+(-1)^n mathrm{Li}_n(e^{-2pi i x})=-(2pi i)^n B_n(x)/n!$, so $I=pi^2 B_2(x/(2pi))$.
    $endgroup$
    – Pierpaolo Vivo
    Jan 25 '16 at 11:09












  • $begingroup$
    @PierpaoloVivo. Please make an answer. As I said, I was stuck and then use brute force. Thanks.
    $endgroup$
    – Claude Leibovici
    Jan 25 '16 at 11:15










  • $begingroup$
    @PierpaoloVivo that's great, thanks. I can rest assured now.
    $endgroup$
    – Kappie001
    Jan 25 '16 at 11:35
















$begingroup$
This identity can be seemingly shown rigorously using the identity between $mathrm{Li}_2$ and Bernoulli polynomials en.wikipedia.org/wiki/Polylogarithm $mathrm{Li}_n(e^{2pi i x})+(-1)^n mathrm{Li}_n(e^{-2pi i x})=-(2pi i)^n B_n(x)/n!$, so $I=pi^2 B_2(x/(2pi))$.
$endgroup$
– Pierpaolo Vivo
Jan 25 '16 at 11:09






$begingroup$
This identity can be seemingly shown rigorously using the identity between $mathrm{Li}_2$ and Bernoulli polynomials en.wikipedia.org/wiki/Polylogarithm $mathrm{Li}_n(e^{2pi i x})+(-1)^n mathrm{Li}_n(e^{-2pi i x})=-(2pi i)^n B_n(x)/n!$, so $I=pi^2 B_2(x/(2pi))$.
$endgroup$
– Pierpaolo Vivo
Jan 25 '16 at 11:09














$begingroup$
@PierpaoloVivo. Please make an answer. As I said, I was stuck and then use brute force. Thanks.
$endgroup$
– Claude Leibovici
Jan 25 '16 at 11:15




$begingroup$
@PierpaoloVivo. Please make an answer. As I said, I was stuck and then use brute force. Thanks.
$endgroup$
– Claude Leibovici
Jan 25 '16 at 11:15












$begingroup$
@PierpaoloVivo that's great, thanks. I can rest assured now.
$endgroup$
– Kappie001
Jan 25 '16 at 11:35




$begingroup$
@PierpaoloVivo that's great, thanks. I can rest assured now.
$endgroup$
– Kappie001
Jan 25 '16 at 11:35


















draft saved

draft discarded




















































Thanks for contributing an answer to Mathematics Stack Exchange!


  • Please be sure to answer the question. Provide details and share your research!

But avoid



  • Asking for help, clarification, or responding to other answers.

  • Making statements based on opinion; back them up with references or personal experience.


Use MathJax to format equations. MathJax reference.


To learn more, see our tips on writing great answers.




draft saved


draft discarded














StackExchange.ready(
function () {
StackExchange.openid.initPostLogin('.new-post-login', 'https%3a%2f%2fmath.stackexchange.com%2fquestions%2f1626131%2fhow-to-do-sum-k-1-infty-frac-coskxk2%23new-answer', 'question_page');
}
);

Post as a guest















Required, but never shown





















































Required, but never shown














Required, but never shown












Required, but never shown







Required, but never shown

































Required, but never shown














Required, but never shown












Required, but never shown







Required, but never shown







Popular posts from this blog

Mario Kart Wii

What does “Dominus providebit” mean?

Antonio Litta Visconti Arese